Which of the following represents a one to one function?
A.Students to thier height
B.Students to their weight
C.Students to their LRN
D.Students to their age​

Answers

Answer 1

Students to their LRN represent a one-to-one function.

If there is exactly one y in the range for every x in the domain of a function f, then that function is a one-to-one function. The range has no y equivalent to more than one x in the domain.

To put it another way, a function is referred to as a "one-to-one" function if various elements of A have different images in B. That is, there are various outputs for various inputs.

Here, each student has a different LRN or Learner Reference Number. No two students will have the same LRN. Therefore, this function is a one-to-one function. The correct option is C.

Two or more students can have the same height, weight, or age. Therefore, the incorrect options are options A, B, and D.

To know more about one-to-one function:

https://brainly.com/question/18154364

#SPJ4


Related Questions

y <= -15x + 3000y <= 5xIn the xy plane, if a point with coordinates (a,b) lies in the solution set of the system of inequalities above, what is the maximum possible value for b?

Answers

B can have a maximum value of 750.

Given info,

y ≤ - 15x + 3000

y ≤ 5x

What is the highest possible value of b in the XY plane if a point with the coordinates (a,b) is found in the system of inequalities above?

The maximum value of b occurs when taking equality.

⇒ - 15x + 3000 = 5x

⇒ - 20x = - 3000

⇒ - 20x = - 3000

⇒ x = 150

Therefore, y = 5 * 150

                    = 750

Hence, the maximum possible value of b = 750

To learn more about equality click here:

brainly.com/question/9070018

#SPJ4

evaluate the expression: v ⋅ w given the vectors: r = <8, 8, -6>; v = <3, -8, -3>; w = <-4, -2, -6> (2 points)

Answers

The product of the vectors v and w is gotten as; v ⋅ w = 22

What is the product of the vectors?

We are given the vectors;

r = <8, 8, -6>

v = <3, -8, -3>

w = <-4, -2, -6>

We want to evaluate v.w which is a product of both vectors.

The formula will be;

v.w = (v_x * w_x) + (v_y * w_y) + (v_z * w_z)

Plugging in the values and solving gives;

v.w = (3 * -4) + (-8 * -2) + (-3 * -6)

v.w = -12 + 16 + 18

v.w = 22

We can conclude that the vector expressions has a product of 22

Read more about Vector Products at; https://brainly.com/question/9131049

#SPJ1

One of the 55 students are selected at random.written as a fraction what is the probability

Answers

If one of the 55 students is selected at random, the probability of this occurring can be written as a fraction by dividing the number of selected students by the total number of students. In this case, the probability would be 1/55.

-3x+y=2
-x+y-4=0
solving a system of linear equations by substitution

Answers

The solution to the system of linear equations by substitution is (1, 5)

How to solve the system of linear equations by substitution

From the question, we have the following parameters that can be used in our computation:

-3x+y=2

-x+y-4=0

Rewrite as

-3x + y = 2

-x + y - 4 = 0

Make y the subject in -x + y - 4 = 0

So, we have

y = x + 4

Substitute y = x + 4 in -3x + y = 2

-3x + x + 4 = 2

Evaluate the like terms

-2x = -2

So, we have

x = 1

Substitute x = 1 in y = x + 4

y = 1 + 4

Evaluate

y = 5

Hence, the solution is  x = 1 and y = 5

Read more about system of equations at

https://brainly.com/question/13729904

#SPJ1

Tanya has a collection of 52 rocks that she wants to put into display cases. Each display case can hold 8 rocks. Tanya divides to find how many cases
she needs.
52÷8=614
How many display cases will Tanya need?

A. 4

B. 6

C. 7

D. 8

E. 10

Answers

The answer would be (B

6 suppose a die is tossed 1000 times, and the following frequencies are obtained for the number of pips up when the die comes to a rest. x1 x2 x3 x4 x5 x6 163 178 142 150 183 184 using the chi-squared goodness of fit test, assess whether we have evidence that this is not a symmetrical die. record the standardized residuals

Answers

We have evidence that this is not a symmetrical die the standardized residuals is 9.5699.

Given that,

The following frequencies are found for the number of pips up when the die comes to rest, assuming a die is tossed 1000 times. Find out if there is proof that this die is not symmetrical using the chi-squared goodness of fit test. The standardized residuals should be noted.

x₁           x₂           x₃           x₄         x₅           x₆

163        178        142        150       183        184

We know that,

          observed       expected         Op-Ep         (Op-Ep)²/Ep

x₁          163                 166.7                -3.7               0.082

x₂          178                 166.7                 11.3              0.7659

x₃          142                 166.7                -24.7            3.6598

x₄          150                 166.7                -16.7             1.673

x₅          183                 166.7                16.3               1.5938

x₆           184                 166.7               17.3               1.7954

                                                                                 -----------

                                                                                 9.5699

Expected = 1/nΣXa=1/6(1000)=166.7

Therefore, We have evidence that this is not a symmetrical die the standardized residuals is 9.5699.

To learn more about standard visit: https://brainly.com/question/23907081

#SPJ4

3. What is the power output of an engine that does 60,000 J of work in 10 s?

Answers

the answer to the problem is 6 KILA watts

Important Formula:

[tex]p=\dfrac{w}{t}[/tex]

Where p is power(measured in watts), w is work(measured in joules), and t is time(measured in seconds).

[tex]w=60,000J[/tex]

[tex]t=10s[/tex]

[tex]p=?[/tex]

____________

[tex]p=\dfrac{w}{t}[/tex]

____________

[tex]p=\dfrac{60,000}{10}[/tex]

____________

[tex]\fbox{p = 6,000 watts}[/tex]

When practicing statistics in real life, it is not very important to check the necessary assumptions of a statistical procedure in order to effectively carry out and use the results. True False

Answers

The given statement is FALSE.

Given statement;

When practicing statistics in real life, it is not very important to check the necessary assumptions of a statistical procedure to effectively carry out and use the results.

The above-mentioned statement is FALSE.

→ We are aware that checking assumptions is crucial when applying statistics in daily life because failing to do so will prevent us from receiving fair answers to our statistical queries or drawing valid inferences. To use the good process and appropriate statistical distributions while utilizing statistics in daily life, it is crucial to examine assumptions.

To learn more about statistics click here:

brainly.com/question/23091366

#SPJ4

If the discriminant of an equation is negative, which of the following is true of the equation.

Answers

Equations have two complex solutions if their discriminant is negative. So, option A is the correct answer.

In algebra, the discriminant of any equation is a quantity that is used to find out about the nature of the roots using the formula = b²-4ac. It tells us if there are no solutions, one solution, or two.

Here, we have been told that the discriminant is negative.

From the formula, we can say that = D = b²-4ac < 0.

Here, if D<0, then x becomes an imaginary value. This means that the roots are going to be imaginary i.e., complex. Thus, the two roots that we get as a result of the formula are both going to be complex.

So, option A is the answer.

The complete that you might be looking for is given below.

If the discriminant of an equation is negative, which of the following is true of the equation

A. it has two complex solutions

B. it has one real solution

C. it had two real solutions

Learn more about the Discriminant on

https://brainly.com/question/15884086?referrer=searchResults

#SPJ4

Answer:

a.two complex solutions

Step-by-step explanation:

A sprinter completed a 100 m run with a time of 12.31 seconds.
Her time,
t
, has been rounded to the nearest hundredth of a second.
Write the error interval for
t
in the form
a

t
<
b
.

Answers

The error interval for the time spent to complete the race is 12.305 ≤ t < 12.315

How to determine the error interval for the time spent to complete the race

From the question, we have the following parameters that can be used in our computation:

Distance = 100 m

Time = 12.31 seconds

From the question, we understand that

Time = rounded to the nearest hundredth of a second.

The highest number that can be rounded to 12.31 to the nearest hundredth is 12.314

While the least is 12.305

When represented as an interval, we have

12.305 ≤ t < 12.314

Add/subtract the error

So, we have

12.305 - 0.001 ≤ t < 12.314 + 0.001

Evaluate

12.305 ≤ t < 12.315

Hence, the error interval is 12.305 ≤ t < 12.315

Read more about error margin at

https://brainly.com/question/25613341

#SPJ1

(1 point) For each of the following pairs of groups G1, G2, determine the number of elements in the direct product G, x G2 of the given order (D6 is the dihedral group of order 12). G G2knumber of elements in G x G2 of order k Z16 Z19 4 U(14) U(13)6

Answers

The number of elements in the direct product are 2.

What is direct product of groups?

The direct product, often written as G x H, is an operation used in group theory, a branch of mathematics, to create a new group from two existing ones. This operation, one of several crucial direct product concepts in mathematics, is the group-theoretic equivalent of the Cartesian product of sets.

Let, the number of elements of order 1 in  [tex]Z_1_6[/tex]  and  [tex]Z_1_9[/tex]  is 1.

Number of elements of order 2 in [tex]Z_1_6[/tex]  and [tex]Z_1_9[/tex]  are 1 and 0 respectively.

Number of elements of order 4 in [tex]Z_1_6[/tex]  and  [tex]Z_1_9[/tex] are 2 and 0 respectively.

Consider the table of number of elements of order.

                           Number of elements of order

                              1         2          4

           [tex]Z_1_6[/tex]              1         1           2

          [tex]Z_1_9[/tex]               1         0          0

Let x ∈ [tex]Z_1_6[/tex] x [tex]Z_1_9[/tex]

O(x) = lcm (O(a), O(b))

where x = (a, b)

That is, a ∈ [tex]Z_1_6[/tex]  and  b ∈ [tex]Z_1_9[/tex]  

Here only (4, 1) satisfies it and [tex]Z_1_6[/tex] has 2 elements of order 4 and [tex]Z_1_9[/tex]  has one element of order 1.

So,  [tex]Z_1_6[/tex] x [tex]Z_1_9[/tex] has 2 x 1 = 2 elements of order 4.

Hence, the number of elements in the direct product are 2.

To know more about direct product of groups, click on the link

https://brainly.com/question/17012757

#SPJ4

Complete question:

Complete question is attached below.

Which statements are true about box plots? Check all that apply.
They help describe sets of data.
They include the mean.
They show the data split into four parts.
They show outliers through really short “whiskers.”
Each section of a box plot represents 25% of the data.

Merry Christmas!

Answers

The true statements are they help describe sets of data, they show the data split into four parts and each section of a box plot represents 25% of the data.

What is the box-and-whisker plot?

A box and whisker plot displays a "box" with its left edge at Q₁, right edge at Q₃, "center" at Q₂ (the median), and "whiskers" at the maximum and minimum.

The box plot is also called box-and-whisker plot.

The data is divided into sections by box plots, with each section containing around 25% of the total data.

Box plots are useful as:

They offer a visual overview of the data,

making it simple for researchers to spot skewness,

the dispersion of the data set.

The minimum value, first quartile, median, third quartile, and maximum value are the five values that make up a box plot.

And they don't include mean.

From the above discussion,

we have three statements that are true and those statements are;

they help describe sets of data, they show the data split into four parts and each section of a box plot represents 25% of the data.

Therefore, the correct statements are A, C and E.

To learn more about the box-and-whisker plot;

https://brainly.com/question/2742784

#SPJ1

Amanda is spreading new grass seed on her yard. The area of her yard is 4,995 square feet. The instructions on the bags of grass seed say that one bag of seed should be used for every 100 square yards. How many bags of grass seed should Amanda use?
Group of answer choices

Answers

The number of bags of grass seed that Amanda should use to spread her yard is 16.65 bags, meaning she needs to buy not more than 17 bags, using mathematical operations.

What are mathematical operations?

The basic mathematical operations we use are subtraction, addition, division, and multiplication.

In this situation, we use the division operation.

The division operation involves four parts:

The dividend (numerator or the number being divided).The divisor (denominator or the number dividing).The quotient (the result of the division operation).The equal symbol (that shows that two values are equal).

The area of the yard = 4,995 ft²

1 yard = 3 feet

Area of the yard = 1,665 square yards(4,995/3)

The number of bags per 100 square yards = 1 bag

For 1,665 square, the number of bags = 16.65 bags (1,665 ÷ 100)

Thus, Amanda requires 16.65 bags of the grass seed for her yard.

Learn more about mathematical operations at https://brainly.com/question/20628271

#SPJ1

Help Please...
Most adult medication doses are for a person weighing 150 pounds. For a 45-pound child, the adult dose should be multiplied by 0.3. If a child's dose of a decongestant is 9 milligrams, what is the adult dose? Please show your work.

Answers

The adult dose for the decongestant is 2.7 milligrams.

How to calculate the adult dose?

The information can be illustrated through an expression. Expression simply refers to the mathematical statements which have at least two terms which are related by an operator and contain either numbers, variables, or both. Addition, subtraction, multiplication, and division are all possible mathematical operations.

Since for a 45-pound child, the adult dose should be multiplied by 0.3, When the child's dose of a decongestant is 9 milligrams, the adult dose will be:

= Children dose × 0.3

= 9 × 0.3

= 2.7

Learn more about expressions on;

brainly.com/question/723406

#SPJ1

In how many ways can Tim draw a red or a green marble from a jar containing 20 red marbles and 4 green marbles?

Answers

The correct answer to how many different ways he can draw is 24.

TRUE/FALSE. incidence of multiple chronic conditions is connected to frailty, a condition marked by a lack of resilience to physiological changes and an elevated risk of poor health outcomes.

Answers

The given

Incidence of multiple chronic conditions is connected to frailty, a condition marked by a lack of resilience to physiological changes and an elevated risk of poor health outcomes.

It is TRUE.

"Information available from the question:"

In the question:

Incidence of multiple chronic conditions is connected to frailty, a condition marked by a lack of resilience to physiological changes and an elevated risk of poor health outcomes.

It is TRUE / FALSE

Now, According to the question:

Let's know:

What is Frailty?

Frailty is most often defined as an aging-related syndrome of physiological decline, characterized by marked vulnerability to adverse health outcomes.

Incidence of multiple chronic conditions is connected to frailty, a condition marked by a lack of resilience to physiological changes and an elevated risk of poor health outcomes.

Hence, The above statement is TRUE.

Learn more about Frailty at:

https://brainly.com/question/28287310

#SPJ4

if x= -1 is a root of x3+3x2-x-3=0 use synthetic division to factor the polynomial completely

Answers

The factors of the polynomial x³+3x²-x-3 are (x+1)(x-1)(x+3).

What are factors?

A factor is a number that divides another number, leaving no remainder.

Given is a polynomial, x³+3x²-x-3

x = -1 is the root of the polynomial, which it will completely divide the polynomial x³+3x²-x-3,

On dividing, we get,

(x³+3x²-x-3)÷(x+1) = (x²+2x-3)

Factorizing, (x²+2x-3), we get,

(x-1)(x+3)

Hence, The factors of the polynomial x³+3x²-x-3 are (x+1)(x-1)(x+3).

For more references on factors, click;

https://brainly.com/question/14067703

#SPJ1

a chain of sport shops catering to beginning skiers, headquartered in aspen, colorado, plans to conduct a study of how much a beginning skier spends on his or her initial purchase of equipment and supplies. based on these figures, it wants to explore the possibility of offering combinations, such as a pair of boots and a pair of skis, to induce customers to buy more. a sample of 44 cash register receipts revealed these initial purchases: $ 180 $ 186 $ 130 $ 263 $ 258 $ 159 $ 127 $ 84 $ 208 180 104 233 92 223 176 216 219 209 205 150 241 110 239 124 173 174 264 232 182 261 216 193 196 223 239 176 261 150 245 84 232 152 95 233 click here for the excel data file required: a. arrive at a suggested class interval. (round your answer up to the next multiple of 5.) b. organize the data into a frequency distribution using a lower limit of $70.

Answers

(a) Class interval of a given data is 36.

(b) Frequency distribution is 44.

We have given that,

total frequency = 44

lower limit = $70

number of class = 5

(a) we have to find class interval,

Class interval =  [tex]\frac{Max. number - min. number}{no. of classes}[/tex]

here maximum number is 264 and minimum number is 84

Class interval = [tex]\frac{264 - 84}{5}[/tex]

                      = [tex]\frac{180}{5}[/tex]

Class interval = 36

we have to round our value to the next multiple of 5

so the class interval will be 40.

(b) A frequency distribution is the pattern of frequencies of a variable. It is the number of times each possible value of a variable occurs in a dataset. There are three types of frequency distribution such as Cumulative frequency distribution, Relative frequency distribution, Relative cumulative frequency distribution.  

The frequency distribution is as follows:

Class interval           Frequency

 70 - 110                         6

 111 - 151                          5

152 - 192                       10

193 - 233                       14

234 -  274                       9

Total                              44

This is the organize data into a frequency distribution.

To know more about class interval here

https://brainly.com/question/28367164

#SPJ4


A 3-gallon container of window cleaner costs $17.76. What is the price per quart?

Answers

Answer: $1.48

Step-by-step explanation:

find how much each gallon cost

1. 17.76/3=5.92

Divide by 4 because 4 quarts equal a gallon

2. 5.92/4=1.48

Therefore each quart equals $1.48,

Hope this helps!

point (9,1) and (7,R) are on a line with a slope of -3/4 what is R

Answers

The value of R is 2.5 or (5/2) . We can find the answer by using concepts of slope of line.

What is the slope of a line?

Slope of a line measures its steepness . Mathematically, its calculated as "rise over run".

When two points are given,

Slope of a line = (y2 - y1)/(x2 - x1)  

Here, y2 = R

          y1 = 1

         x2 = 7

          x1 = 9

Slope of a line = (R-1)/(7-9)

In the que, its given slope  = -3/4

So, (R-1)/(-2) = -3/4

      R = 5/2 or 2.5

Learn more about slope of a line using given link:
https://brainly.com/question/16949303

#SPJ1

Given the equation, proof it

Answers

Answer:

Step-by-step explanation:

I forgot to add the last reason is AAS (two angles one side are congruent = triangles are congruent)

S.

1.Given

2. AE = EC

3. <AEB = <CED

4. <CDE = <ABE

5. triangles congruent

R.

1. Given

2. Segment bisector theorem

3. Vertical angles theorem

4. PAI Theorem

Hope this helps!

A tile is randomly selected from a bag that contains 10 tiles. The tiles are either blue, green, red, or yellow. After the color is recorded, the tile is placed back in the bag. A tile is pulled 100 times, of which 24 are blue tiles, 12 are green tiles, and 12 are red tiles.

Answers

The number of tiles in the bag of 10 that are yellow in colour are; 5 tiles

How to find the probability of selection?

We are given;

Total number of tiles in bag = 10 tiles

Now, we are told that in the bag, the tiles are either blue, green, red, or yellow.

Now, after selecting a tile, we are told that it is put back and as such we have after 100 selections that we have 24 blue tiles, 12 green tiles and 12 red tiles. Thus;

Number that will be yellow = 100 - 24 - 12 - 12 = 52 yellow tiles

Thus, it means that the probability of selecting a yellow tile is;

52/100

Thus, number of tiles out of the 10 in the bag that are yellow =

(52/100) * 10 = 5.2

Approximately 5 yellow tiles

Read more about probability of selection at; https://brainly.com/question/251701

#SPJ1

Complete question is;

How many yellow tiles are most likely in the bag? Enter the answer in the box. A tile is randomly selected from a bag that contains 10 tiles. The tiles are either blue, green, red, or yellow. After the color is recorded, the tile is placed back in the bag. A tile is pulled 100 times, of which 24 are blue tiles, 12 are green tiles, and 12 are red tiles.​

find the absolute maximum and absolute minimum of f (x, y) among points in the triangle with vertices (0, 0), (1, 0), and (1, 1).

Answers

The absolute maximum and absolute minimum of f(x,y) = 3 + xy -x - 2y is f(0,0) and f(1,1) respectively.

The absolute maximum point is a point where the function obtains its greatest possible value.

And Absolute minimum is a point where the function obtains its least possible value. This is the smallest value that a mathematical function can have over its entire curve.

Given equation is, f(x,y) = 3 + xy - x - 2y

we will check absolute maximum and absolute minimum of f(x,y) at the points (0,0), (1,0), (1,1).  

f(0,0) = 3 + (0)(0) - 0 - 2(0) = 3

f(1,0) = 3 + (1)(0) - 1 - 2 (0) = 2

f(1,1) = 3 + (1)(1) - 1 - 2(1) = 1

Therefore we get absolute maximum at f(0,0) and absolute minimum at f(1,1).

An absolute minimum also called a global minimum, occurs when a point of the function is lower any other point on the function within the function's domain. A local minimum also called relative minimum occurs when a point is lower than the points surrounding it.

Given question is incomplete. Complete question is:

find the absolute maximum and absolute minimum of f (x, y) = 3 + xy - x -2y  among points in the triangle with vertices (0, 0), (1, 0), and (1, 1).

To know more about maximum here

https://brainly.com/question/13176730

#SPJ4

I need help with this please help me

Answers

It’s -1,0, since it’s only moving one placement backwards, and there is no movement up and down

If X is a metric space, and fn:X→R (n∈N) is a sequence of functions, then fn converges pointwise to f if for every x∈X one has limn→[infinity]fn(x)=f(x).

Answers

|f(x) − f n(x)| = lim m→∞

|f m(x) − f n(x)| ≤ ϵ /2< ϵ

Converges Uniformly

A sequence of functions f n(x); n = 1, 2, 3,…. Is said to be uniformly convergent to f for a set E of values of x, if for each ε > 0, a positive integer N exists such that |f n(x) – f(x)| < ε for n ≥ N and x ∈ E.

A sequence (f n) of functions f n : A → R is uniformly Cauchy

on A if for every ϵ > 0 there exists N ∈ N such that

m, n > N implies that |f m(x) − f n(x)| < ϵ for all x ∈ A.

The key part of the following proof is the argument to show that a pointwise

convergent, uniformly Cauchy sequence converges uniformly.

A sequence (f n) of functions f n : A → R converges uniformly on

A if and only if it is uniformly Cauchy on A.

Suppose that ( f n) converges uniformly to f on A. Then, given ϵ > 0, there

exists N ∈ N such that

|f n(x) − f(x)| < ϵ/2

for all x ∈ A if n > N.

It follows that if m, n > N then

|f m(x) − f n(x)| ≤ |f m(x) − f(x)| + |f(x) − f n(x)| < ϵ for all x ∈ A,

which shows that (f n) is uniformly Cauchy.

Conversely, suppose that (f n) is uniformly Cauchy. Then for each x ∈ A, the

real sequence (f n(x)) is Cauchy, so it converges by the completeness of R. We

define f : A → R by

f(x) = limn→∞

f n(x),

and then f n → f pointwise.

To prove that f n → f uniformly, let ϵ > 0. Since (f n) is uniformly Cauchy,

can choose N ∈ N (depending only on ϵ) such that

|f m(x) − f n(x)| < ϵ /2

for all x ∈ A if m, n > N.

Let n > N and x ∈ A. Then for every m > N we have

|f n(x) − f(x)| ≤ |f n(x) − f m(x)| + |f m(x) − f(x)| < ϵ /2 + |f m(x) − f(x)|.

Since f m(x) → f(x) as m → ∞, we can choose m > N (depending on x, but it

doesn’t matter since m doesn’t appear in the final result) such that

|f m(x) − f(x)| < ϵ/2.

It follows that if n > N, then

|f n(x) − f(x)| < ϵ for all x ∈ A,

Then proves that f n → f uniformly

Alternatively, we can take the limit as m → ∞ in the Cauchy condition to get

for all x ∈ A and n > N that

|f(x) − f n(x)| = lim m→∞

|f m(x) − f n(x)| ≤ ϵ /2< ϵ

To learn more about Converges Uniformly visit:

brainly.com/question/29736068

#SPJ4

what was the mistake that was made?

Answers

Answer: they didnt distribute the 7 to the 4.5

Step-by-step explanation:

when distributing something outside of a parenthesis, you have to distribute it to all terms within the parenthesis, not just the variable

the point (-4,-6,10) lies on the surface of a sphere with centre (2,3,-1). hence find the volume of the sphere

Answers

The radius of the sphere with the given point and the center is √238.

Here we have to find the radius of the sphere.

Given data:

The point on the surface of a sphere is (-4, -6, 10) and the point on the center is (2, 3, -1).

The formula for the radius:

r² = (x2 - x1)² + (y2 - y1)² + (z2 - z1)²

where (x1, y1, z1) = point on the center

(x2, y2, z2) = point on the sphere

Now putting the values we have

r² = (-4-2)² + ( -6-3)² + ( 10 +1)²

  =( -6)² +(- 9)²  + (11)²

  = 36 + 81 + 121

  = 238

r = √238.

Therefore the radius is √ 238.

To know more about the sphere refer to the link given below:

https://brainly.com/question/22807400

#SPJ4

Find the area of the triangle. [?] units² Round to the nearest hundredth.​

Answers

The area of the triangle is 48 square units.

To calculate the length of each side of the triangle ACB, the radii of the circles should be added respectively.

Length of side BC = 9+3 = 12 units

Length of side CA = 3+5 = 8 units

Length of side BA = 9+5 = 14 units

Area of triangle = (1/2 × Base × Height) square units

Area of triangle ACB = 1/2 × CA× BC sq units

                          Area = 1/2 × 8 × 12 = 48 sq units

Hence, the area of triangle ACB is 48 square units.

Learn more about the area of triangles here:

https://brainly.com/question/28884904

Answer:

47.91 units²

Step-by-step explanation:

The radius of each circle is:

Circle A:  radius = 5Circle B:  radius = 9Circle C:  radius = 3

Therefore, the side lengths of the triangle are:

AB = 5 + 9 = 14BC = 9 + 3 = 12AC = 3 + 5 = 8

Heron's Formula allows us to find the area of a triangle in terms of its side lengths.

[tex]\boxed{\begin{minipage}{8 cm}\underline{Heron's Formula}\\\\$A=\sqrt{s(s-a)(s-b)(s-c)}$\\\\where:\\ \phantom{ww}$\bullet$ $A$ is the area of the triangle. \\ \phantom{ww}$\bullet$ $a, b$ and $c$ are the side lengths of the triangle. \\ \phantom{ww}$\bullet$ $s$ is half the perimeter.\\\end{minipage}}[/tex]

The perimeter of a two-dimensional shape is the distance all the way around the outside. Therefore, half the perimeter of the triangle is:

[tex]\implies s=\dfrac{14+12+8}{2}=17[/tex]

To calculate the area of the triangle, substitute the sides lengths and the value of s into Heron's formula and solve for A:

[tex]\implies A=\sqrt{17(17-14)(17-12)(17-8)}[/tex]

[tex]\implies A=\sqrt{17(3)(5)(9)}[/tex]

[tex]\implies A=\sqrt{2295}[/tex]

[tex]\implies A=47.906158268...[/tex]

[tex]\implies A=47.91\; \sf units^2\;\;(nearest\;hundredth)[/tex]

A pharmacy has determined that a healthy person should receive 70 units of
proteins, 100 units of carbohydrates and 20 units of fat daily. If the store carries
the six types of health food with their ingredients as shown in the table below,
what blend of foods satisfies the requirements at minimum cost to the
pharmacy? Make a mathematical model for the given problem.
Foods Protein units Carbohydrates
units
Fat units Cost per unit
A 20 50 4 2
B 30 30 9 3
C 40 20 11 5
D 40 25 10 6
E 45 50 9 8
F 30 20 10 8

Answers

optimal solution is arrived with value of variables as :

x1=0.9091,x2=1.8182,x3=0,x4=0,x5=0,x6=0

minimize cost:

z=2x₁+3x₂+5x₃+6x₄+8x₅+8x₆=2x

subject to:

20x₁+30x₂+40x₃+40x₄+45x₅+30x₆ ≥70 : amount of protein

50x₁+30x₂+20x₃+25x₄+50x₅+20x₆ ≥100 : amount of carbohydrate

4x₁+9x₂+11x₃+10x₄+9x₅+10x₆ ≥20 : amount of fat

where x₁, x₂, x₃, x₄, x₅, x₆ are units of 6 foods

solution using Simplex method:

After introducing artificial variables:

Min Z=2x₁+3x₂+5x₃+6x₄+8x₅+8x₆+0S₁+0S₂+0S₃+MA₁+MA₂+MA₃

subject to

20x₁+30x₂+40x₃+40x₄+45x₅+30x₆-S₁+A₁ ≥ 70

50x₁+30x₂+20x₃+25x₄+50x₅+20x₆-S₂+A₂≥ 100

4x₁+9x₂+11x₃+10x₄+9x₅+10x₆-S₃+A₃=20

and x₁,x₂,x₃,x₄,x₅,x₆,A₁,A₂,A₃≥0

Refer to the image for Z

Since all Zj-Cj≤0

Hence, optimal solution is arrived with value of variables as :

x1=0.9091,x2=1.8182,x3=0,x4=0,x5=0,x6=0

Min Z=7.2727

Learn more about Simplex Method at:

https://brainly.com/question/14410494

#SPJ1

The UGA Journalism Department is interesting in determining if there is a significant difference in the number of hours, on average per week, that UGA males and UGA female students spend reading any online or printed newspaper. Randomly selected students at UGA were asked how many hours per week they read any online or printed newspaper, their age, and gender. The 95% confidence interval for reading newspapers on μF-urn is (-1.1, 1.3). 1) Interpret this confidence interval. Because 0 is not in this interval, we cannot conclude that there is a difference in newspaper reading, on average, for females and males. Because 0 is not in this interval, we can conclude that there is a difference in newspaper reading, on average, for females and males. Because 0 is in this interval, we cannot conclude that there is a difference in newspaper reading, on average, for females and males. Because 0 is in this interval, we can conclude that there is a difference in newspaper reading, on average, for females and males.

Answers

The option C, “because 0 is in this interval, we cannot conclude that there is a difference in newspaper reading, on average, for females and males” is correct.

In the given question, the UGA Journalism Department is interesting in determining if there is a significant difference in the number of hours, on average per week, that UGA males and UGA female students spend reading any online or printed newspaper.

Randomly selected students at UGA were asked how many hours per week they read any online or printed newspaper, their age, and gender. The 95% confidence interval for reading newspapers on μ(F)-μ(M) is (-1.1, 1.3).

1) We have to interpret this confidence interval. The given options are:

• Because 0 is not in this interval, we cannot conclude that there is a difference in newspaper reading, on average, for females and males.

• Because 0 is not in this interval, we can conclude that there is a difference in newspaper reading, on average, for females and males.

• Because 0 is in this interval, we cannot conclude that there is a difference in newspaper reading, on average, for females and males.

• Because 0 is in this interval, we can conclude that there is a difference in newspaper reading, on average, for females and males.

Since there interval was given.

The given interval for the 95% confidence interval for reading newspapers on μ(F)-μ(M) is (-1.1, 1.3).

Since the value of interval is between -1.1 and 1.3. So 0 is in this interval. So we cannot conclude that there is a difference in newspaper reading, on average, for females and male.

So the option C, “because 0 is in this interval, we cannot conclude that there is a difference in newspaper reading, on average, for females and males” is correct.

To learn more about confidence interval link is here

brainly.com/question/29657446

#SPJ4

Other Questions
At temperatures near room temperature, the temperature dependence of the conductivity for intrinsic germanium is found to be given by o = CT-3/2 exp(- 2KT where C is a temperature-independent constant and T is in Kelvins. Using the above equation to calculate the intrinsic electrical conductivity of germanium at 175 C gains on the sale of marketable securities and inventory, or any gain that is due to section 1245 depreciation recapture is ineligible for recognition under the The graph of the basic cubic function h(x)=x is shown. Suppose thatk(x)=-h(x-1)+2. If point A at (2, 8) undergoes this transformation to become pointA', what are the coordinates of point A'? One justification for the new social and economic order of the North was based ona. Marxism.b. Social Darwinism.c. Utilitarian doctrine.d. None of the above Underlying the dividend irrelevance theory proposed by Miller and Modigliani is their argument that the value of the firm is determined only by its basic earning power and its business risk. True or False the governor of texas can use the veto in various ways to reject acts by the legislature, and different types of vetoes have specific features. label each description with the type of veto that it best and most closely describes. Gallop is a finding in which body system O Gait O Respiratory O CardiovascularO Neuro What is biodiversity discuss the value of biodiversity What are main causes of biodiversity loss?; What is a naturally occurring factor that could lead to a loss of biodiversity?; What factors in the environment affect the biodiversity of organisms in an area?; What are two specific human activities that result in a loss of biodiversity? which two acronyms represent the data link sublayers that ethernet relies upon to operate? (choose two.) Assume 185 and 122 are unsigned 8-bit decimal integers. Calculate 185 122. Is there overflow, underflow, or neither? your neighborhood gym tracked how many days each of its members worked out over the past week. let g A line passes through the points (14, 10) and (0, 2). What is its equation in slope-intercept form? bruno latour maintains that it would make no sense to say ramses ii died of tuberculosis because the ancient egyptians had not identified tuberculosis as a disease. Suppose f(x) = 12x - 1,400 and g(x) = 12x - 1,600. Describe the transformation of the graph of f(x) to the graph of g(x). Group of answer choicesthe graph is translated left 200 units.the graph is translated up 200 units.the graph is translated right 200 units.the graph is translated down 200 units Think of a historical example, other than slavery, where the is-ought fallacy was used to support a negative practice or custom.Subject- Logic(but it wasn't listed in the "Pick a subject" tab) Which statement is true about the rights of U.S. citizens?a . Men have more rights than women.b .The rights of U.S. citizens are guaranteed by the Constitution.c . Some accused individuals cannot have their trials decided by a jury of peers.d . Citizens have the right to refuse to pay taxes. Sarah purchased 8kg of sugar, 10kg offlour, 500g of cocoa, 225g of pecans,and 275g of coconut. How much do allher groceries weigh in kilograms? one of the limitations of body mass index (bmi) as a method of assessing overweight and obesity is that it does not differentiate between fat and lean body mass. True or False? Anyone know how to solve this? You are supposed to find X Y and Z within the circle. a loudspeaker diaphragm is oscillating in simple harmonic motion with a frequency of 440 hz and a maximum displacement of 0.75 mm. what are the (a) angular frequency, (b) maximu